Physics Olympiad Problem

Two identical blocks A and B each of mass M are placed on a long inclined plane (angle of inclination = θ ) with A higher up than B. The coefficients of friction between the plane and the blocks A and B are respectively µ A and µ B with tan θ > µ B > µ A. The two blocks are initially held fixed at a distance d apart. At t = 0 the two blocks are released from rest.

Find the time in which the two blocks collide?

My answer ; - tCollision=2dtanθ(sinθμAcosθ)g(sinθμBcosθ)\large{t_{Collision} = \sqrt{\dfrac{2d \tan \theta ( \sin \theta - \mu_A \cos\theta)}{g (\sin \theta - \mu_B \cos\theta)}}}

What about you?Is my answer correct?

I am not sure with my answer, please all of us, lets discuss!

#Mechanics

Note by Md Zuhair
4 years, 2 months ago

No vote yet
1 vote

  Easy Math Editor

This discussion board is a place to discuss our Daily Challenges and the math and science related to those challenges. Explanations are more than just a solution — they should explain the steps and thinking strategies that you used to obtain the solution. Comments should further the discussion of math and science.

When posting on Brilliant:

  • Use the emojis to react to an explanation, whether you're congratulating a job well done , or just really confused .
  • Ask specific questions about the challenge or the steps in somebody's explanation. Well-posed questions can add a lot to the discussion, but posting "I don't understand!" doesn't help anyone.
  • Try to contribute something new to the discussion, whether it is an extension, generalization or other idea related to the challenge.
  • Stay on topic — we're all here to learn more about math and science, not to hear about your favorite get-rich-quick scheme or current world events.

MarkdownAppears as
*italics* or _italics_ italics
**bold** or __bold__ bold

- bulleted
- list

  • bulleted
  • list

1. numbered
2. list

  1. numbered
  2. list
Note: you must add a full line of space before and after lists for them to show up correctly
paragraph 1

paragraph 2

paragraph 1

paragraph 2

[example link](https://brilliant.org)example link
> This is a quote
This is a quote
    # I indented these lines
    # 4 spaces, and now they show
    # up as a code block.

    print "hello world"
# I indented these lines
# 4 spaces, and now they show
# up as a code block.

print "hello world"
MathAppears as
Remember to wrap math in \( ... \) or \[ ... \] to ensure proper formatting.
2 \times 3 2×3 2 \times 3
2^{34} 234 2^{34}
a_{i-1} ai1 a_{i-1}
\frac{2}{3} 23 \frac{2}{3}
\sqrt{2} 2 \sqrt{2}
\sum_{i=1}^3 i=13 \sum_{i=1}^3
\sin \theta sinθ \sin \theta
\boxed{123} 123 \boxed{123}

Comments

The two blocks will meet up after a time tt such that

d+g2(sin(θ)μBcos(θ))=g2(sin(θ)μAcos(θ))d + \dfrac{g}{2}(\sin(\theta) - \mu_{B}\cos(\theta)) = \dfrac{g}{2}(\sin(\theta) - \mu_{A}\cos(\theta)),

which when solved for tt gives us tc=2dgcos(θ)(μBμA)t_{c} = \sqrt{\dfrac{2d}{g\cos(\theta)(\mu_{B} - \mu_{A})}}.

Suppose μA=μB\mu_{A} = \mu_{B}. Then you wouldn't expect the two blocks to ever meet, but your formula would have them meeting after 2dtan(θ)g\sqrt{\dfrac{2d\tan(\theta)}{g}} seconds, while mine would have tCt_{C} \to \infty as expected.

Brian Charlesworth - 4 years, 1 month ago

Log in to reply

This seems to be a generalization of that "underkill" problem that was posted recently.

Steven Chase - 4 years, 1 month ago

Log in to reply

Exactly. I found it a bit odd at first that cos(θ)\cos(\theta) was in the denominator, since this would result in a finite tct_{c} for θ=0\theta = 0, which doesn't make sense, (although an infinite tct_{c} for θ=π/2\theta = \pi/2 does make sense as the two blocks would both be in free-fall so A would never catch up). But then I remembered that we require that

tan(θ)>μBθ>arccos(11+μB2)\tan(\theta) \gt \mu_{B} \Longrightarrow \theta \gt \arccos\left(\dfrac{1}{\sqrt{1 + \mu_{B}^{2}}}\right).

Brian Charlesworth - 4 years, 1 month ago

Log in to reply

@Brian Charlesworth It would be interesting to consider this same problem with a curved surface and friction.

Steven Chase - 4 years, 1 month ago

Log in to reply

@Steven Chase Okay! So go on, Can you post one?

Md Zuhair - 4 years, 1 month ago

Log in to reply

@Md Zuhair I'll look into it

Steven Chase - 4 years, 1 month ago

Which underkill problem? Can you please provide the link sir?

Md Zuhair - 4 years, 1 month ago

Log in to reply

@Md Zuhair https://brilliant.org/problems/mechanics-overkill-2/?ref_id=1349100

Steven Chase - 4 years, 1 month ago

Log in to reply

@Steven Chase Thank you,

Md Zuhair - 4 years, 1 month ago

One can generalise further : If the time at which they first meet is t1t_{1} , the subsequent times of meeting again (after collision) turns out to be odd multiples of t1t_{1}

Rohith M.Athreya - 4 years, 1 month ago

Log in to reply

@Rohith M.Athreya Hey! Whats your JEE Main score?

Md Zuhair - 4 years, 1 month ago

Log in to reply

@Md Zuhair around 290 :(

Rohith M.Athreya - 4 years, 1 month ago

Log in to reply

@Rohith M.Athreya Well, lets see your picture in newspaper!

Md Zuhair - 4 years, 1 month ago

Log in to reply

@Md Zuhair why?not at all!

my friends are expecting around 310-315

Rohith M.Athreya - 4 years, 1 month ago

Log in to reply

@Rohith M.Athreya Okay, leave that! You will surely qualify JEE Advanced with rank above 0.5k this year brother! I have seen your brain!

Md Zuhair - 4 years, 1 month ago

Log in to reply

@Md Zuhair well....

I'll take your word for it(if i write advanced)

thank you!

Rohith M.Athreya - 4 years, 1 month ago

Log in to reply

@Rohith M.Athreya If you dont write advanced, ill delete my Brilliant account!

Md Zuhair - 4 years, 1 month ago

@Rohith M.Athreya So @Rohith M.Athreya are you giving either of ISI or CMI ?

Aditya Narayan Sharma - 4 years, 1 month ago

Log in to reply

@Aditya Narayan Sharma CMI yes

II no

Rohith M.Athreya - 4 years, 1 month ago

Thanks sir!

Md Zuhair - 4 years, 1 month ago

Even I got the same answer as you sir. :)

@Md Zuhair From where did you get this problem?

Ankit Kumar Jain - 4 years, 1 month ago

Log in to reply

i did a very fine mistake! Its correct now!

Md Zuhair - 4 years, 1 month ago

@Md Zuhair Yup, my answer matches with the one given by Brian Charlesworth and yes, its correct :-) ! Sorry yours was wrong :-( .

Ayon Ghosh - 4 years, 1 month ago

Log in to reply

Ya I got it now

Md Zuhair - 4 years, 1 month ago

Let me tag some of you,

@Aniket Sanghi , @Rohith M.Athreya , @Ayon Ghosh , @Steven Chase , @Brian Charlesworth!

Md Zuhair - 4 years, 2 months ago

Log in to reply

http://olympiads.hbcse.tifr.res.in/olympiads/wp-content/uploads/2017/01/INPhO2017-Solution-20170131.pdf

Rohith M.Athreya - 4 years, 1 month ago
×

Problem Loading...

Note Loading...

Set Loading...